THMMY.gr

Μαθήματα Βασικού Κύκλου => Συστήματα Αυτομάτου Ελέγχου II - Παλιά Θέματα => Topic started by: Αλντεμπαράν on January 21, 2017, 18:17:31 pm



Title: [ΣΑΕ ΙΙ] Θέματα 2017
Post by: Αλντεμπαράν on January 21, 2017, 18:17:31 pm
Ανεβάζω κάποια λυμένα που βρήκα! Αν ο συγγραφέας τους θέλει copyright, ας μου στείλει πμ. Μέχρι τότε τον ευχαριστούμε !

https://mega.nz/#F!PQphFJTS!MSoaRblmIT7IfWVPJgr1Jw (https://mega.nz/#F!PQphFJTS!MSoaRblmIT7IfWVPJgr1Jw)

edit: fix link


Title: Re: [ΣΑΕ ΙΙ] Θέματα 2017
Post by: Andromedas on May 30, 2017, 03:28:53 am
Άντε να ξεκινάμε και φέτος
θέμα 1 α) Φεβρουάριος 2017
/εδιτ Παίζει να έχω λάθος όπως μου είπε ένας συνάδελφός. Θα το ελέγξω και θα το διορθώσω
/edit2 έχω λάθος
(1/(α*(|b|+c))) δεν είναι <= από το 1/(α*|b|)+1/(α*c) για όσους το κατέβασαν  το κατεβάζω και θα προσπαθήσω να το λύσω


Title: Re: [ΣΑΕ ΙΙ] Θέματα 2017
Post by: raptalex on May 30, 2017, 22:10:31 pm
Άντε να ξεκινάμε και φέτος
θέμα 1 α) Φεβρουάριος 2017

Βασικά επειδή είναι ΠΑΡΑ πολλά τα pages αν μπορεί κάποιος να μαζέψει τις ανεβασμένες λύσεις ώστε να μπορούμε να τις σχολιάζουμε πιο εύκολα... Το λέω βασικά μήπως κάποιος που έχει ξαναδώσει το μάθημα μήπως το έχει κάνει ήδη..


Title: Re: [ΣΑΕ ΙΙ] Θέματα 2017
Post by: nemdam on May 31, 2017, 17:21:26 pm
Άντε να ξεκινάμε και φέτος
θέμα 1 α) Φεβρουάριος 2017

Να κάνω μια ερώτηση; Με το x τι συμβαίνει κατά τον προσδιορισμό της Lyapunov; Γιατί μου φαίνεται κάπου χάνομαι με τις παρενθέσεις και τους κοινούς παράγοντες.


Title: Re: [ΣΑΕ ΙΙ] Θέματα 2017
Post by: Andromedas on May 31, 2017, 17:46:37 pm
Να κάνω μια ερώτηση; Με το x τι συμβαίνει κατά τον προσδιορισμό της Lyapunov; Γιατί μου φαίνεται κάπου χάνομαι με τις παρενθέσεις και τους κοινούς παράγοντες.
Η πρώτη  αλλαγή u=Ax άρα βγαίνει: V'=x*(ax+ βAx+f(x)Ax)=x*x*(a+ βΑ+f (x)Α)
Που δίνει X τετράγωνο.
Κυρίως το ζήτημα είναι ότι το c δεν μπορείς να το εισάγεις στο u οπότε το χρησιμοποιείς στην lyapunov.


Title: Re: [ΣΑΕ ΙΙ] Θέματα 2017
Post by: gmpougiou on May 31, 2017, 18:47:26 pm
υπαρχει καποιος που ν ελυσε  καποιο θεμα απο την προοδο 2017 κι μπορει ν το ανεβασει ?


Title: Re: [ΣΑΕ ΙΙ] Θέματα 2017
Post by: Apostolof on May 31, 2017, 23:21:47 pm
υπαρχει καποιος που ν ελυσε  καποιο θεμα απο την προοδο 2017 κι μπορει ν το ανεβασει ?

Ο Ρόβι στο μάθημα σήμερα, το θέμα με το sat. Θα το ανεβάσω στις σημειώσεις.


Title: Re: [ΣΑΕ ΙΙ] Θέματα 2017
Post by: Indy on June 01, 2017, 15:01:35 pm
Επίσης, οι λύσεις των περσινών (Φεβρουάριος 17) είναι more than welcome.


Title: Re: [ΣΑΕ ΙΙ] Θέματα 2017
Post by: Andromedas on June 04, 2017, 17:31:04 pm
Ο Ρόβι στο μάθημα σήμερα, το θέμα με το sat. Θα το ανεβάσω στις σημειώσεις.
Δεν το βρήκα στις σημειώσεις που έχεις ανεβάσει μεχρι τώρα. Παίζει να μην τις ανέβασες ακόμη οπότε /εδιτ δική μου λύση θέμα 3.
Δεν είμαι σίγουρος για το Lassale, όχι δεν ότι χρειάζεται, αν αρκεί αυτό που έχω γράψει ή όχι. Αν μπορεί κάποιος να απαντήσει.
Επίσης σε ένα γραμμικό σύστημα το χαρ πολυώνυμο είναι το det(λ*Ι-Α)=0 όπου λ->s. Αρκεί να πεις επειδή οι ρίζες του χαρακτηριστικού πολυωνύμου είναι αρνητικές (έστω το Real αρνητικό) το σύστημα ευσταθές;


Title: Re: [ΣΑΕ ΙΙ] Θέματα 2017
Post by: tpt on June 04, 2017, 18:28:46 pm
Δεν το βρήκα στις σημειώσεις που έχεις ανεβάσει μεχρι τώρα. Παίζει να μην τις ανέβασες ακόμη οπότε θέμα 3.
Δεν είμαι σίγουρος για το Lassale, όχι δεν ότι χρειάζεται, αν αρκεί αυτό που έχω γράψει ή όχι. Αν μπορεί κάποιος να απαντήσει.
Επίσης σε ένα γραμμικό σύστημα το χαρ πολυώνυμο είναι το det(λ*Ι-Α)=0 όπου λ->s. Αρκεί να πεις επειδή οι ρίζες του χαρακτηριστικού πολυωνύμου είναι αρνητικές (έστω το Real αρνητικό) το σύστημα ευσταθές;
η λυση ειναι απο το μάθημα η δικη σ?


Title: Re: [ΣΑΕ ΙΙ] Θέματα 2017
Post by: Andromedas on June 04, 2017, 18:36:12 pm
η λυση ειναι απο το μάθημα η δικη σ?
δική μου... αφού έγραψα ότι δεν είμαι σίγουρος για το επιχείρημα Lassale...


Title: Re: [ΣΑΕ ΙΙ] Θέματα 2017
Post by: tpt on June 04, 2017, 18:44:02 pm
δική μου... αφού έγραψα ότι δεν είμαι σίγουρος για το επιχείρημα Lassale...
σωστα.


Title: Re: [ΣΑΕ ΙΙ] Θέματα 2017
Post by: ablaoublas on June 04, 2017, 18:44:26 pm
Επίσης σε ένα γραμμικό σύστημα το χαρ πολυώνυμο είναι το det(λ*Ι-Α)=0 όπου λ->s. Αρκεί να πεις επειδή οι ρίζες του χαρακτηριστικού πολυωνύμου είναι αρνητικές (έστω το Real αρνητικό) το σύστημα ευσταθές;

Ναι, συγκεκριμένα θα έχεις ασυμπτωτική ευστάθεια.


Title: Re: [ΣΑΕ ΙΙ] Θέματα 2017
Post by: nemdam on June 04, 2017, 19:22:44 pm
Δεν το βρήκα στις σημειώσεις που έχεις ανεβάσει μεχρι τώρα. Παίζει να μην τις ανέβασες ακόμη οπότε /εδιτ δική μου λύση θέμα 3.
Δεν είμαι σίγουρος για το Lassale, όχι δεν ότι χρειάζεται, αν αρκεί αυτό που έχω γράψει ή όχι. Αν μπορεί κάποιος να απαντήσει.
Επίσης σε ένα γραμμικό σύστημα το χαρ πολυώνυμο είναι το det(λ*Ι-Α)=0 όπου λ->s. Αρκεί να πεις επειδή οι ρίζες του χαρακτηριστικού πολυωνύμου είναι αρνητικές (έστω το Real αρνητικό) το σύστημα ευσταθές;

Καλησπέρα.
Από τη στιγμή που έχεις τον πίνακα Α, είναι πιο σίγουρο να μελετήσεις την ευστάθεια παίρνοντας τις ιδιοτιμές του από το det(λI-A).
Εμένα μου βγαίνουν και οι 2 να έχουν αρνητικο πραγματικό μέρος, άρα έχουμε και τοπ ασ ευστάθεια νομίζω.


Title: Re: [ΣΑΕ ΙΙ] Θέματα 2017
Post by: Andromedas on June 04, 2017, 19:28:14 pm
Δεν ήμουν σίγουρος ότι έχεις τ. α. ευστάθεια για αυτό το ρώτησα. Αλλά ναι νομίζω ότι αρκεί.


Title: Re: [ΣΑΕ ΙΙ] Θέματα 2017
Post by: nemdam on June 05, 2017, 08:35:03 am
Έχει λύσει κανείς το 2ο Φεβρουαρίου 2017 ή το 1ο Προοδου 2017; Αυτά με τα κατακόρυφα συστήματα. Με μπερδεύει αρκετά η φυσική στερεών.


Title: Re: [ΣΑΕ ΙΙ] Θέματα 2017
Post by: DarkPassenger on June 06, 2017, 02:49:18 am
Άντε να ξεκινάμε και φέτος
θέμα 1 α) Φεβρουάριος 2017
/εδιτ Παίζει να έχω λάθος όπως μου είπε ένας συνάδελφός. Θα το ελέγξω και θα το διορθώσω
/edit2 έχω λάθος
(1/(α*(|b|+c))) δεν είναι <= από το 1/(α*|b|)+1/(α*c) για όσους το κατέβασαν  το κατεβάζω και θα προσπαθήσω να το λύσω
τελικά το έλυσες?


Title: Re: [ΣΑΕ ΙΙ] Θέματα 2017
Post by: Andromedas on June 06, 2017, 10:15:00 am
τελικά το έλυσες?
Όχι έκανα κύκλους γύρω από την ίδια ιδέα και είπα να το αφήσω καμιά μέρα  :-\


Title: Re: [ΣΑΕ ΙΙ] Θέματα 2017
Post by: DarkPassenger on June 06, 2017, 12:33:04 pm
Όχι έκανα κύκλους γύρω από την ίδια ιδέα και είπα να το αφήσω καμιά μέρα  :-\
υπάρχει κ αυτή (https://www.thmmy.gr/smf/index.php?topic=68281.msg1176878#msg1176878) η λύση πάντως αλλά δεν ξέρω κατά πόσο είναι σωστή.


Title: Re: [ΣΑΕ ΙΙ] Θέματα 2017
Post by: Andromedas on June 06, 2017, 12:56:06 pm
υπάρχει κ αυτή (https://www.thmmy.gr/smf/index.php?topic=68281.msg1176878#msg1176878) η λύση πάντως αλλά δεν ξέρω κατά πόσο είναι σωστή.
δεν ισχύει διότι |f (x)|<=c άρα -c <=f (x)<=c συνεπώς αν το πολλαπλασιασεις με αρνητικό όπως έκανε  βάζοντας u=-x/abs(b) έχεις αντιστροφή της ανισότητας και πάλι θετικό


Title: Re: [ΣΑΕ ΙΙ] Θέματα 2017
Post by: DarkPassenger on June 06, 2017, 13:43:16 pm
δεν ισχύει διότι |f (x)|<=c άρα -c <=f (x)<=c συνεπώς αν το πολλαπλασιασεις με αρνητικό όπως έκανε  βάζοντας u=-x/abs(b) έχεις αντιστροφή της ανισότητας και πάλι θετικό
και επίσης δεν εχω καταλαβει εκεί στην ανισότητα που γράφει "<- lyapunov" αντικατέστησε το |f(x)| με το c..


Title: Re: [ΣΑΕ ΙΙ] Θέματα 2017
Post by: Ap.Mor. on June 07, 2017, 13:05:28 pm
έχει λύσει κάποιος και θέλει να μοιραστεί τα θέματα με τις εξισώσεις κατάστασης και τα ελατήρια του Φεβρουαρίου και της Προόδου του 2017;


Title: Re: [ΣΑΕ ΙΙ] Θέματα 2017
Post by: Luffy on June 07, 2017, 16:03:32 pm
έχει λύσει κάποιος και θέλει να μοιραστεί τα θέματα με τις εξισώσεις κατάστασης και τα ελατήρια του Φεβρουαρίου και της Προόδου του 2017;

Ρίξε μια ματιά στο παράδειγμα 11.4 στο Khalil nonlinear systems. Είναι ουστιαστικά το θέμα του Φλεβάρη. Ελπίζω να σε βοηθήσει.



Title: Re: [ΣΑΕ ΙΙ] Θέματα 2017
Post by: Επίδοξος on June 07, 2017, 16:17:07 pm
έχει λύσει κάποιος και θέλει να μοιραστεί τα θέματα με τις εξισώσεις κατάστασης και τα ελατήρια του Φεβρουαρίου και της Προόδου του 2017;

Αν δεν κάνω λάθος είναι:

m1 q1'' = - k1 q1 - b1 q1' + b1 q2' + u1

m2 q2'' = - k2 q2 - b1 q2' + b1 q1' + u2

(για την πρόοδο)


Title: Re: [ΣΑΕ ΙΙ] Θέματα 2017
Post by: Ap.Mor. on June 07, 2017, 16:38:58 pm
Ρίξε μια ματιά στο παράδειγμα 11.4 στο Khalil nonlinear systems. Είναι ουστιαστικά το θέμα του Φλεβάρη. Ελπίζω να σε βοηθήσει.


Ευχαριστώ πολύ!
Για όποιον ψάχνει το βιβλίο
https://www.researchgate.net/file.PostFileLoader.html?id=532f014bd2fd64df198b45ef&assetKey=AS%3A272534484783104%401441988692243 (https://www.researchgate.net/file.PostFileLoader.html?id=532f014bd2fd64df198b45ef&assetKey=AS%3A272534484783104%401441988692243)


Title: Re: [ΣΑΕ ΙΙ] Θέματα 2017
Post by: nemdam on June 07, 2017, 17:18:16 pm
Αν δεν κάνω λάθος είναι:

m1 q1'' = - k1 q1 - b1 q1' + b1 q2' + u1

m2 q2'' = - k2 q2 - b1 q2' + b1 q1' + u2

Τα u1 και u2 πού ακριβώς είναι;


Title: Re: [ΣΑΕ ΙΙ] Θέματα 2017
Post by: Επίδοξος on June 07, 2017, 17:32:45 pm
Ξέχασα να το γράψω πάνω, αυτά είναι για την πρόοδο του 17. Τα u οι εξωτερικές δυνάμεις που λέει η εκφώνηση είναι.


Title: Re: [ΣΑΕ ΙΙ] Θέματα 2017
Post by: orestisf on June 08, 2017, 13:32:27 pm
Θ6 Ιουνης 17. Βρήκατε καμία Lyapunov  που να δουλεύει;


Title: Re: [ΣΑΕ ΙΙ] Θέματα 2017
Post by: ablaoublas on June 08, 2017, 13:40:24 pm
Θ6 Ιουνης 17. Βρήκατε καμία Lyapunov  που να δουλεύει;

Επισυνάπτω την λύση μου για το 6β και ακούω γνώμες


Title: Re: [ΣΑΕ ΙΙ] Θέματα 2017
Post by: Andromedas on June 08, 2017, 14:00:01 pm
Θ6 Ιουνης 17. Βρήκατε καμία Lyapunov  που να δουλεύει;
κανονική
Γενικά έβγαινε σχετικά γρήγορα αν πήγαινες σε τετραγωνική μορφή και έβγαζες αρνητικά ορισμένο.
επισυνάπτω την λύση μου.


Title: Re: [ΣΑΕ ΙΙ] Θέματα 2017
Post by: Apostolof on June 08, 2017, 14:01:41 pm
Τα υπόλοιπα θέματα (εκτός από κάποια πράγματα στα 3α, 4β).

Εδιτ: στο 4δ είναι V'<0 και αρνητικά ορισμένη (όχι ημιορισμένη).


Title: Re: [ΣΑΕ ΙΙ] Θέματα 2017
Post by: ablaoublas on June 08, 2017, 14:11:28 pm
κανονική
Γενικά έβγαινε σχετικά γρήγορα αν πήγαινες σε τετραγωνική μορφή και έβγαζες αρνητικά ορισμένο.
επισυνάπτω την λύση μου.

Το Μ2 τι παριστάνει ; Τον αριθμό στο (1,2)  του Q ; Θετικό δεν βγαίνει ;


Α σόρρυ λάθος, η Μ2 είναι η ορίζουσα άρα είναι αρνητικά ορισμένος ο πίνακας άρα κομπλε... Νομίζω σωστός είσαι, δες λίγο και την δική μου λύση και πες μου αν είναι σωστή και αυτή  :P


Title: Re: [ΣΑΕ ΙΙ] Θέματα 2017
Post by: Andromedas on June 08, 2017, 14:15:17 pm
Το Μ2 τι παριστάνει ; Τον αριθμό στο (1,2)  του Q ; Θετικό δεν βγαίνει ;
υπό ορίζουσα
Μ1 υπο ορίζουσα Μ1=(1,1)
Μ2 δεύτερη υπό ορίζουσα  Μ2=det(του πίνακα)
αν είναι και οι δυο αρνητικές είναι αρνητικά ορισμένος ο πίνακας


Title: Re: [ΣΑΕ ΙΙ] Θέματα 2017
Post by: kostas15 on August 23, 2017, 23:22:32 pm
3o στα θέματα του ιουνίου του 2017 μπορεί να μου πει κάποιος πως υπολογίζω τον πίνακα μετασχηματισμού για να πάω στην ελέγξιμη κανονική μορφή;

Βασικά δεν ξέρω πως θα υπολογίσω τον πίνακα ελεγξιμότητας του μετασχηματισμένου συστήματος, αυτού δλδ που το ονομάζει με περισπωμένη στις σημειώσεις του ροβιθάκη.

Οποιαδήποτε βοήθεια ευπρόσδεκτη!

EDIT:η απορία λύθηκε ! (σελ 40 σημειώσεις exomag)


Title: Re: [ΣΑΕ ΙΙ] Θέματα 2017
Post by: heavy melon on September 09, 2017, 23:23:03 pm
Ιουνης 2017, επισυναπτω τη λυση μου! περιμενω διορθωσεις  :P
στην αιτιολογηση, βαλατε κατι τετοιο ή οτι η ταξη του συστηματος ειναι ιση με τον αριθμο των μεταβλητων καταστασης που επιλεγω στο 2ο ερωτημα?


Title: Re: [ΣΑΕ ΙΙ] Θέματα 2017
Post by: tzitzikas1 on September 09, 2017, 23:39:55 pm
Ιουνης 2017, επισυναπτω τη λυση μου! περιμενω διορθωσεις  :P
στην αιτιολογηση, βαλατε κατι τετοιο ή οτι η ταξη του συστηματος ειναι ιση με τον αριθμο των μεταβλητων καταστασης που επιλεγω στο 2ο ερωτημα?

Εγώ απλά είχα γράψει ότι είναι τρίτης τάξης επειδή η πρώτη διαφορική είναι δεύτερης τάξης και η δεύτερη πρώτης τάξης!
Oπότε το σύστημα είναι το άθροισμά τους δλδ τρίτης τάξης και μου το πήρε για σωστό αν κρίνω από το βαθμό μου και ότι δεν είχα γράψει και πολλά  :D


Title: Re: [ΣΑΕ ΙΙ] Θέματα 2017
Post by: tzitzikas1 on September 10, 2017, 01:22:18 am
Ioύνιος 17 4β με γραμμικοποίηση βγαίνει ασταθές στη γειτονιά του 0;  ΜΕ τον πίνακα Α=[κ*κ κ*κ
                                                                                                                                            -κ*κ κ*κ]

Σωστά η λάθος; 


Title: Re: [ΣΑΕ ΙΙ] Θέματα 2017
Post by: Perasmus on September 11, 2017, 13:01:47 pm
Θέμα 2 -  Ιούνιος 2017

Στο α) αν πας να υπολογίσεις Μ = [Β  ΑΒ  Α2Β  Α3Β] δεν παίζει με την καμία να χρειάζεται να κάνεις τόσες πράξεις.
Αν υπολογίσεις όμως τις 2 πρώτες στήλες του Μ βγαίνουν  [0,0,0,3] και [0,0,0,-6] , που είναι γραμμικά εξαρτημένες αφού η 2η προκύπτει αν πολλ/σιάσεις την 1η με το -2.
Άρα rank(M)<n  (όπου n=4 η τάξη του συστήματος), αφου rank(πίνακα) = ο αριθμός των γραμ. ανεξ. στηλών του.
Οπότε το α) σύστημα :   ΜΗ ΕΛΕΓΞΙΜΟ.
Είναι σωστό το σκεπτικό μου;;  :)

Στο β) , ισχύει ότι ο πίνακας Α είναι στην Ελεγξιμη κανονική μορφή;;


Title: Re: [ΣΑΕ ΙΙ] Θέματα 2017
Post by: kokkinos drakos ths zhxal on September 12, 2017, 17:15:50 pm
ιουνιος 2017 θεμα πρωτο. οι μεταβλητες καταστασεις που θα παρω θα ειναι x1=x,x2=x',x3=i ή θα παρω x1=x',x2=i? γιατι παρακατω ρωταει και την ταξη του συστηματος


Title: Re: [ΣΑΕ ΙΙ] Θέματα 2017
Post by: bag9 on September 13, 2017, 17:43:44 pm
Παιδιά μια απορία, γιατί υπήρχε και στα θέματα του Ιουνίου 2017. Όταν σου δίνει πίνακα C, της μορφής [1 0] τότε ο ελεγκτής περιέχει μόνο τον όρο x1, δηλαδή u= -k1*x1.
Αντίστοιχα αν ήταν [0 1] τότε ο ελεγκτής θα ήταν u= -k2*x2
Αυτό μου το είπε ο Ροβιθάκης στο γραπτό.Τότε όμως γιατί στις σημειώσεις υπάρχει αυτό στο συνημμένο?!!


Title: Re: [ΣΑΕ ΙΙ] Θέματα 2017
Post by: kokkinos drakos ths zhxal on September 13, 2017, 18:01:17 pm
Παιδιά μια απορία, γιατί υπήρχε και στα θέματα του Ιουνίου 2017. Όταν σου δίνει πίνακα C, της μορφής [1 0] τότε ο ελεγκτής περιέχει μόνο τον όρο x1, δηλαδή u= -k1*x1.
Αντίστοιχα αν ήταν [0 1] τότε ο ελεγκτής θα ήταν u= -k2*x2
Αυτό μου το είπε ο Ροβιθάκης στο γραπτό.Τότε όμως γιατί στις σημειώσεις υπάρχει αυτό στο συνημμένο?!!

Μηπως ο ρομπι αναφεροταν σε ελεγκτη εξοδου? Τον ιουνιο τουλαχιστον ηταν ελεγκτης εξοδου, μπορει στο παραδειγμα να θελει ελεγκτη εισοδου


Title: Re: [ΣΑΕ ΙΙ] Θέματα 2017
Post by: bag9 on September 13, 2017, 18:08:50 pm
Μηπως ο ρομπι αναφεροταν σε ελεγκτη εξοδου? Τον ιουνιο τουλαχιστον ηταν ελεγκτης εξοδου, μπορει στο παραδειγμα να θελει ελεγκτη εισοδου
? Δεν το έπιασα...
Ο ελεγκτής στο θέμα 5 του 2017 είναι ακριβώς η περίπτωση που λεω, με C=[0 1]. Αν έβαζες μέσα στον u, -k1*x1 στο κοβε όλο.


Title: Re: [ΣΑΕ ΙΙ] Θέματα 2017
Post by: persephonee on September 13, 2017, 18:49:02 pm
? Δεν το έπιασα...
Ο ελεγκτής στο θέμα 5 του 2017 είναι ακριβώς η περίπτωση που λεω, με C=[0 1]. Αν έβαζες μέσα στον u, -k1*x1 στο κοβε όλο.
στο θέμα 5 ζητάει γραμμικό ελεγκτή ανάδρασης εξόδου (y). ο ελεγκτής -k1*x1 είναι γραμμική ανάδραση της κατάστασης 1. Το παράδειγμα που έχεις επισυνάψει πιο πάνω είναι δυναμική ανάδραση καταστάσεων. Είναι τρία διαφορετικά πράγματα...


Title: Re: [ΣΑΕ ΙΙ] Θέματα 2017
Post by: zwitsa on September 14, 2017, 15:22:49 pm
στο θέμα 5 ζητάει γραμμικό ελεγκτή ανάδρασης εξόδου (y). ο ελεγκτής -k1*x1 είναι γραμμική ανάδραση της κατάστασης 1. Το παράδειγμα που έχεις επισυνάψει πιο πάνω είναι δυναμική ανάδραση καταστάσεων. Είναι τρία διαφορετικά πράγματα...

οκ τώρα μπερδεύτηκα. Τι είναι ο γραμμικός ελεγκτής ανάδρασης εξόδου;;;  ^seestars^


Title: Re: [ΣΑΕ ΙΙ] Θέματα 2017
Post by: zwitsa on September 14, 2017, 16:03:26 pm
Οταν πηγα να δω το γραπτο μ ειπε οτι μπερδεψα γραμμικο ελεγκτη καταστασεων και εξοδου.
Μπορει καποιος να διευκρινισει τη διαφορα?

λοιπόν στον ιούνιο 2017 η έξοδος μας είναι ίση με χ2. άρα ελεγκτή παίρνεις u=-kx2. αντικαθιστάς στις εξισώσεις κατάστασεις και ξαναφτιάχνεις πίνακα Α' και από αυτόν βρίσκεις χαρ. πολυώνυμο.


Title: Re: [ΣΑΕ ΙΙ] Θέματα 2017
Post by: dimikara on September 14, 2017, 19:13:50 pm
Ιούνης 17 στο 3α πως ακριβώς βρίσκουμε τον Τ?

Επίσης, στο 4β ποιά παίρνουμε ως H που έχει κάποιος στις λύσεις του πιο πίσω?

εδιτ: για το 3α έχει παρόμοιο στις σημειώσεις exomag σελ40


Title: Re: [ΣΑΕ ΙΙ] Θέματα 2017
Post by: vag178 on September 14, 2017, 21:30:46 pm
Ιούνης 17 στο 3α πως ακριβώς βρίσκουμε τον Τ?

Επίσης, στο 4β ποιά παίρνουμε ως H που έχει κάποιος στις λύσεις του πιο πίσω?

εδιτ: για το 3α έχει παρόμοιο στις σημειώσεις exomag σελ40

για τ Τ υπαρχει σ προηγουμενα ποστ (σελ: 40 σειμειωσεις τ exomag νμζω)

αν καταλαβα σωστα τ F ειναι η πανω εξισωση dot(χ1) κ η Η dot(χ2) δλδ τ Α B C D θα ειναι οι τιμες του πινακα Α(διαφορετικος απο τ προηγουμενο) οπου (dot(z)=Az)

μ καθε επιφυλαξη


Title: Re: [ΣΑΕ ΙΙ] Θέματα 2017
Post by: s0r0n on September 15, 2017, 11:31:23 am
Ποιος ηταν ο ελεγκτης στο 2β????? (Σεπτ 2017)


Title: Re: [ΣΑΕ ΙΙ] Θέματα 2017
Post by: kostas15 on September 15, 2017, 11:56:01 am
Στο 2α πηρε κανεις δυναμικη αναδραση; (Σεπτ 2017)


Title: Re: [ΣΑΕ ΙΙ] Θέματα 2017
Post by: Niobe on September 15, 2017, 12:13:29 pm
Ποιος ηταν ο ελεγκτης στο 2β?????

Δε μπορω να τον βγαλω. Φτανω σε σημειο που πρεπει να παρω περιπτωσεις για το γ αλλα δε βγαζει πουθενα


Title: Re: [ΣΑΕ ΙΙ] Θέματα 2017
Post by: Niobe on September 15, 2017, 12:52:24 pm
Ποιος ηταν ο ελεγκτης στο 2β?????

Το βρηκα. Σελ 31/38 απο τις φετινες σημειωσεις part 2 εχει ιδιο παραδειγμα σχεδον.

Εδιτ: στην ουσια δε βρισκεις k1 k2 αλλα το διαστημα στο οποιο βρισκεται το κερδος μιας εξτρα ενισχυτικης βαθμιδας για να ειναι τα k1 k2 μικροτερα απο 1 και να πληροιται και το α ερωτημα


Title: Re: [ΣΑΕ ΙΙ] Θέματα 2017
Post by: australopithecus on September 15, 2017, 19:40:42 pm
το συστημα ειναι ελαχιστης υλοποιησης αν ειναι ταυτοχρονα ελεγξιμο και παρατηρησιμο σωστα;


Title: Re: [ΣΑΕ ΙΙ] Θέματα 2017
Post by: abadasa13 on September 15, 2017, 21:28:12 pm
το συστημα ειναι ελαχιστης υλοποιησης αν ειναι ταυτοχρονα ελεγξιμο και παρατηρησιμο σωστα;

Ναι! Διαφάνεια 6 (Linear Output Feedback Control) τελευταία σελίδα.
Τι καλά που θα ήταν να το έγραφα κιόλας! :P


Title: Re: [ΣΑΕ ΙΙ] Θέματα 2017
Post by: australopithecus on September 15, 2017, 21:38:59 pm
Ποιος ηταν ο ελεγκτης στο 2β?????
απτο (α) ειχες κ1=4/γ και κ2=4/γ*(1-1/γ)
εβαζες κκ1=4/γ και κκ2=4/γ*(1-1/γ), αρα κ1=4/κγ<1 και κ2=4/κγ*(1-1/γ)<1
αρα κ>4/γ και κ>4/γ*(1-1/γ), δηλαδη κ>4/γ, κ1, κ2 οπως πριν
(σελ 31-32 μερος 2ο σημειωσεις αποστολοφ)


Ναι! Διαφάνεια 6 (Linear Output Feedback Control) τελευταία σελίδα.
Τι καλά που θα ήταν να το έγραφα κιόλας! :P
θενκς!


Title: Re: [ΣΑΕ ΙΙ] Θέματα 2017
Post by: heavy melon on September 15, 2017, 22:38:22 pm
Ναι! Διαφάνεια 6 (Linear Output Feedback Control) τελευταία σελίδα.
Τι καλά που θα ήταν να το έγραφα κιόλας! :P

αν πεις όταν οι ιδιοτιμές ταυτίζονται με τους πόλους της συνάρτησης μεταφοράς;
no hope  ^innocent^


Title: Re: [ΣΑΕ ΙΙ] Θέματα 2017
Post by: heavy melon on September 15, 2017, 22:39:39 pm
απτο (α) ειχες κ1=4/γ και κ2=4/γ*(1-1/γ)
εβαζες κκ1=4/γ και κκ2=4/γ*(1-1/γ), αρα κ1=4/κγ<1 και κ2=4/κγ*(1-1/γ)<1
αρα κ>4/γ και κ>4/γ*(1-1/γ), δηλαδη κ>4/γ, κ1, κ2 οπως πριν
(σελ 31-32 μερος 2ο σημειωσεις αποστολοφ)

θενκς!

ok μπορεί να το είχε κάνει, όμως όταν έχεις διαβάσει 100 σελίδες σημειώσεις & θεωρία & παλιά θέματα, serioulsy, θα πρεπε να το θυμάσαι?  ^banghead^


Title: Re: [ΣΑΕ ΙΙ] Θέματα 2017
Post by: lopodyths on September 15, 2017, 22:52:51 pm
στο 3β σου ζητουσε να κανεις το συστημα γραμμικο και ελεγξιμο.Το 1ο  σχετικα ευκολα βαζοντας το μη γραμμικο στοιχειο στον ελεγκτη ωστε να απαλοιφθει,ομως το 2ο wtf? εφοσον εξαρχης εχεις ενα μη γραμμικο συστημα και δεν μπορεις να τσεκαρεις ελεγξιμοτητα,μετα το κανεις γραμμικο χρησιμοποιωντας τον ελεγκτη αρα τι σκατα(ηδη το ελεγξες)?Το μονο που μπορω να σκεφτω ειναι ο αρχικος ελεγκτης να εχει εναν δευτερο ελεγκτη μεσα(u' πχ) και να τσεκαρες μετα την ελεγξιμοτητα(που ειχες πινακες Α,Β) που προφανως εβγαινε αλλα και αυτο πολυ τρολλ ερωτημα


Title: Re: [ΣΑΕ ΙΙ] Θέματα 2017
Post by: Indy on September 16, 2017, 16:43:13 pm
Στα δύο τελευταία ερωτήματα του πρώτου θέματος έγραψα θέλει δύο ενεργοποιητές (όσες κ οι μεταβλητές κατάστασης) και δύο αισθητήρες (ένας για είσοδο κ ένας για έξοδο).. Πιάνω τίποτα;


Title: Re: [ΣΑΕ ΙΙ] Θέματα 2017
Post by: Indy on September 18, 2017, 15:09:33 pm
Κανείς ρε παιδιά;


Title: Re: [ΣΑΕ ΙΙ] Θέματα 2017
Post by: tpt on September 18, 2017, 16:12:33 pm
Κανείς ρε παιδιά;
ετσι βγαζεις οτι ελαχιστο πληθος ενεργοποιητων και αισθητήρων  ειναι 1.


Title: Re: [ΣΑΕ ΙΙ] Θέματα 2017
Post by: Indy on September 18, 2017, 16:38:53 pm
ετσι βγαζεις οτι ελαχιστο πληθος ενεργοποιητων και αισθητήρων  ειναι 1.

Thanks


Title: Re: [ΣΑΕ ΙΙ] Θέματα 2017
Post by: greekoo on January 14, 2018, 11:49:08 am
Θέμα 2 Πρόοδος 2017:

Η σωστή απάντηση είναι το (2) ? Διότι αφού είμαι πάνω στον κύκλο θα συνεχίσω να κινούμαι πάνω σε αυτόν;


Title: Re: [ΣΑΕ ΙΙ] Θέματα 2017
Post by: greekoo on January 21, 2018, 12:41:32 pm
Έχει ασχοληθεί κανείς με τον περιβόητο Φλεβάρη του 17;

Το 1α είναι λυμένο στις σημειώσεις (σελ 88-89 απ'το merged pdf του Apostolof), όπου εκεί είχαμε την γενική περίπτωση με πίνακες.

Στο 1β όμως πώς θα το προσεγγίσουμε αφού δεν είναι γνωστές οι παράμετροι α, β ;

Τέλος, το θέμα 2 είναι παρμένο από το παράδειγμα 11.4 από το βιβλίο Khali Non-Linear Systems και η λύση του φαίνεται από άλλο πλανήτη. Ορίζει μάλιστα κάτι σταθερές του αυτοκινήτου όπως φυσική συχνότητα του λαστίχου, του σώματος του στηρίγματος κλπ και κάνει κάτι scalings με σταθερές 'ε' κλπ.
Not sure if όντως ήθελε να γράψουμε όλα αυτά (που τότε το θέμα είναι όντως ότι να ναι) ή απλώς ήθελε να γράψουμε τις απλές εξισώσεις και να σταματήσουμε εκεί...


Title: Re: [ΣΑΕ ΙΙ] Θέματα 2017
Post by: Alex_corfu on January 21, 2018, 19:32:54 pm
Θέμα 3 Σεπτέμβριος 2017 έχει κανείς καμία ιδέα? Ή ακόμα καλύτερα να ανεβάσει καμία λύση? Ευχαριστώ εκ των προτέρων.


Title: Re: [ΣΑΕ ΙΙ] Θέματα 2017
Post by: persephonee on January 21, 2018, 20:29:14 pm
Θέμα 3 Σεπτέμβριος 2017 έχει κανείς καμία ιδέα? Ή ακόμα καλύτερα να ανεβάσει καμία λύση? Ευχαριστώ εκ των προτέρων.
Δυστυχώς δεν έχω χρόνο να το λύσω αλλά αυτό που θυμάμαι ότι είχα κάνει στο α) (και μου το πήρε σωστό) είναι:
Αρχικά θέτεις x1=y x2=y_dot και με βάση την (3) βρίσκεις τις δύο εξισώσεις κατάστασης. Από αυτές βρίσκεις το σημείο ισορροπίας (μοναδικό), το οποίο βγαίνει διάφορο του (0,0) οπότε κάνεις αλλαγή μεταβλητής για να το φέρεις στο (0,0).
Τώρα με βάση το νέο σύστημα παίρνεις κάποια υποψήφια Lyapunov για να δείξεις την ευστάθεια. Θυμάμαι ότι αν πάρεις την κλασσική V=1/2(x1^2+x2^2) δεν μπορείς να βγάλεις κάποιο συμπέρασμα για το πρόσημο της V_dot οπότε προσθέτεις ή αφαιρείς κάτι στην V και η V_dot θα προκύψει αρνητικά ημιορισμένη.
Μέχρι εδώ έχεις δείξει ότι το σημείο ισορροπίας είναι ολικά ευσταθές. Για να αποδείξεις την ασυμπτωτική ευστάθεια πας με Lasalle και δείχνεις ότι το μεγαλύτερο αμετάβλητο σύνολο είναι το σημείο ισορροπίας.
Αφού το (0,0) είναι μοναδικό ολικά ασυμπτωτικά ευσταθές ΣΙ, και το ΣΙ του αρχικού συστήματος (πριν την αλλαγή μεταβλητής) θα είναι μοναδικό ολικά ασυμπτωτικά ευσταθές.
Τώρα για το β), βρίσκεις τις εξισώσεις κατάστασης του συστήματος και θέτεις u τέτοιο ώστε να φεύγει το απόλυτο που είναι η μη γραμμικότητα. Από εκεί και πέρα δεν ξέρω τι άλλο ήθελε...
Ελπίζω να βοήθησα κάπως.


Title: Re: [ΣΑΕ ΙΙ] Θέματα 2017
Post by: princess_of_the_dawn on January 22, 2018, 04:14:08 am
Θέμα 3 Σεπτέμβριος 2017 έχει κανείς καμία ιδέα? Ή ακόμα καλύτερα να ανεβάσει καμία λύση? Ευχαριστώ εκ των προτέρων.
Επισύναψα μια πιθανή λύση αλλά δεν είμαι καθόλου σίγουρη α) αν η Λιαπούνοφ είναι σωστή β) αν χρειάζονται τα κ1, κ2 στις κ/σεις χ1 και χ2 του ελεγκτή, οπότε οποιαδήποτε διόρθωση είναι more than welcome.

Αύριο έρχεται κύμα αποριώνε


Title: Re: [ΣΑΕ ΙΙ] Θέματα 2017
Post by: ENtz on January 22, 2018, 15:11:25 pm
Επισύναψα μια πιθανή λύση αλλά δεν είμαι καθόλου σίγουρη α) αν η Λιαπούνοφ είναι σωστή β) αν χρειάζονται τα κ1, κ2 στις κ/σεις χ1 και χ2 του ελεγκτή, οπότε οποιαδήποτε διόρθωση είναι more than welcome.

Αύριο έρχεται κύμα αποριώνε

Εγώ πήρα πιο απλή lyapunov : V = (k/2)*(x1-gM/k)^2 + (M/2)*x2^2 και βγαινει μια χαρα
στο β πηρα u = c2x2|x2| -gM +r (αφου στις ΕΚ προκυπτει u/M) δε βαζω k1 k2 αφου δε σβηνεις τα χ1 χ2 απ την αρχικη, γιατι να βαλεις κ αλλα?



Title: Re: [ΣΑΕ ΙΙ] Θέματα 2017
Post by: princess_of_the_dawn on January 22, 2018, 17:44:31 pm
τα χ1, χ2 δε ξερω απλα σκεφτηκα ότι εφόσον ειναι ελεγκτής τα χρειάζεται αλλά μαλλον περισσευουν
στο cx2|x2| δε θέλει κι ενα Μ στον ελεγκτη; αλλιώς δε φεύγει
επίσης εφόσον σου λέει να ειναι ελέγξιμο δε θα πρεπει να βαλεις κι ενα k3*u ;
αλλιώς πώς θα είναι ελέγξιμο;


Title: Re: [ΣΑΕ ΙΙ] Θέματα 2017
Post by: ENtz on January 22, 2018, 18:28:27 pm
Εκανες ενα λαθακι οταν εγραψες τις εξισωσεις καταστασης, δεν το διαιρεσες με το Μ. Οποτε μετα δε θελει να βαλεις Μ. Το r που εγραψα ειναι η καινουρια εισοδος για να ειναι ελεγξιμο, βαλτο u' αν θες δεν παιζει ρολο ο συμβολισμος. Δε χρειαζεται να εχει καποιο k αυτο


Title: Re: [ΣΑΕ ΙΙ] Θέματα 2017
Post by: princess_of_the_dawn on January 22, 2018, 18:46:16 pm
ναι το ειδα σμρ το λαθακι αλλα θελει διαίρεση με το Μ γιατι στην πραγματικοτητα διαιρειται απλά το ξέχασα :P
για το r/u0 έχεις δίκαιο, αρκεί να έχει κάποια καινούρια είσοδο για να μπορεί να είναι ελέγξιμο, απορώ όμως αν το βάλουμε το κ3 και πούμε ότι πρέπει απλά να είναι !=0 αν θα το θεωρούσε λάθος (στο ίδιο καταλήγουμε :P)
Ροβι είναι οπότε μάλλον θα το θεωρούσε :-\


Title: Re: [ΣΑΕ ΙΙ] Θέματα 2017
Post by: Alex_corfu on January 22, 2018, 22:47:25 pm
Κατάλαβα, ευχαριστώ πολύ, βγαίνει όντως με αυτή τη Luapunov V(z)=(k/2)*(z1)^2+(M/2)*(z2)^2 & ελεγκτή u=c2*x2*|x2|-M*g+Μ*u'.. May the odds be ever in our favor!  ;D


Title: Re: [ΣΑΕ ΙΙ] Θέματα 2017
Post by: princess_of_the_dawn on January 23, 2018, 01:42:38 am
Θέμα 2 Πρόοδος 2017:

Η σωστή απάντηση είναι το (2) ? Διότι αφού είμαι πάνω στον κύκλο θα συνεχίσω να κινούμαι πάνω σε αυτόν;
Ας πει κάποιος τη γνωμη του.

Εγώ θα έλεγα το 2 σίγουρα και το 4(με  δισταγμό). Εφ'όσον η V είναι συνάρτηση Lyap σημαίνει ότι ικανοποιεί την προϋπόθεση να ειναι θετική παντού  εκτός από τα σημεία ισορροπίας* όπου μηδενίζεται και αρνητική παράγωγος σημαίνει ότι η συνάρτηση τείνει προς τα σημεία ισορροπίας. Στο διάγραμμα φαίνεται σημεία ισορροπίας (έστω σ.ι: α,-α,β,-β) να ειναι τα σημεία όπου μηδενίζουν τα x1 &x2 άρα θα θέλουμε η f(x) να "τραβάει" τη συνάρτηση προς τα σημεία αυτά, το οποίο το κάνουν τα διανύσματα (2) και (4). Όχι;;;

*θετικά ορισμένη είναι η συνάρτηση η οποία μηδενίζεται μόνο στο 0 ή μόνο στα σ.ι;;
εδιτ: απαντάω στην εαυτή μου: μάλλον μόνο στα σ.ι απλά κάνουμε συνήθως γραμμικοποίηση γι αυτό θεωρούμε ότι μηδενίζεται στο 0


Title: Re: [ΣΑΕ ΙΙ] Θέματα 2017
Post by: princess_of_the_dawn on January 23, 2018, 03:06:20 am
Παρακαλώ απαντήσετε το συντομότερο δυνατόν κλείνουν σπίτια

1.Όταν έχουμε Lyap συναρτήσει των x1,x2 τρόπον τινά ώστε αυτά τα 2 να πολλαπλασιάζονται στο ένα μέρος, ας πούμε V'=-x2*sinx τότε η συνάρτηση δεν είναι αρνητικά ορισμένη, έτσι? Γενικά απλά λέμε ότι είναι ημιορισμένη και ότι το σημείο ισορροπίας είναι ευσταθές αλλά όχι ασυμπτωτικά ευσταθές ή απλά δεν υπάρχει περίπτωσις πρέπει να βρω άλλη Lyap?


2α. Γενικά αν έχω ένα σ.ι και η d(Lyap)/dt μηδενίζεται στο σ.ι και σε έναν οριακό κύκλο, γίνεται αρνητική εκτός αυτού και θετική εντός,  τότε το σημείο ισορροπίας δεν είναι ασυμπτωτικά ευσταθές,ε? Αφού οι λύσεις τείνουν στον οριακό κύκλο και μόνο αν ξεκινήσω από το σ.ι δε θα μεταβληθώ ε? Άρα είναι απλώς ευσταθές?
2β. Αν σε ίδια φάση η παράγωγος της Lyap γίνεται αρνητική εντός του κύκλου και θετική εκτός, τότε το σ.ι είναι τοπικά (στην περιοχή εντός του κύκλου) ασυμπτωτικά ευσταθές?

γ.  Αν έχω 2 σημεία ισορροπίας, έστω το (0,0) και το (α,0) τότε το (0,0) είναι πάλι απλώς ευσταθές έτσι? Αφού έχουμε μεγαλύτερο αμετάβλητο σύνολο, το (α,0) ή κάτι έχω καταλάβει πολύ λάθος;


Title: Re: [ΣΑΕ ΙΙ] Θέματα 2017
Post by: greekoo on January 23, 2018, 13:39:49 pm
Ερώτηση

Σε πολλές ενδεικτικές λύσεις έχω δει να αγνοείται η βαρύτητα στην κατάστρωση των εξισώσεων (π.χ. Προοδος 17 ή Φλεβ 17). Το σωστό δεν θα ήταν να μην την αγνοήσουμε;

Παρακαλώ απαντήσετε το συντομότερο δυνατόν κλείνουν σπίτια

1.Όταν έχουμε Lyap συναρτήσει των x1,x2 τρόπον τινά ώστε αυτά τα 2 να πολλαπλασιάζονται στο ένα μέρος, ας πούμε V'=-x2*sinx τότε η συνάρτηση δεν είναι αρνητικά ορισμένη, έτσι? Γενικά απλά λέμε ότι είναι ημιορισμένη και ότι το σημείο ισορροπίας είναι ευσταθές αλλά όχι ασυμπτωτικά ευσταθές ή απλά δεν υπάρχει περίπτωσις πρέπει να βρω άλλη Lyap?


2α. Γενικά αν έχω ένα σ.ι και η d(Lyap)/dt μηδενίζεται στο σ.ι και σε έναν οριακό κύκλο, γίνεται αρνητική εκτός αυτού και θετική εντός,  τότε το σημείο ισορροπίας δεν είναι ασυμπτωτικά ευσταθές,ε? Αφού οι λύσεις τείνουν στον οριακό κύκλο και μόνο αν ξεκινήσω από το σ.ι δε θα μεταβληθώ ε? Άρα είναι απλώς ευσταθές?
2β. Αν σε ίδια φάση η παράγωγος της Lyap γίνεται αρνητική εντός του κύκλου και θετική εκτός, τότε το σ.ι είναι τοπικά (στην περιοχή εντός του κύκλου) ασυμπτωτικά ευσταθές?

γ.  Αν έχω 2 σημεία ισορροπίας, έστω το (0,0) και το (α,0) τότε το (0,0) είναι πάλι απλώς ευσταθές έτσι? Αφού έχουμε μεγαλύτερο αμετάβλητο σύνολο, το (α,0) ή κάτι έχω καταλάβει πολύ λάθος;


1) Στις σημειώσεις λέει ότι μια συνάρτηση V είναι αρνητικά ορισμένη αν V > 0 και V(x) != 0 για χ !=0 και V(x) = 0 για χ = 0.
Άρα λογικά το V' = -x2sinx1 δεν είναι αρνητικά ορισμένο αφού μηδενίζεται και για x1 !=0...

2α) Νομίζω πως είναι ασυμπτωτικά ευσταθές

2β) Αν γίνεται θετική εκτός του κύκλου τότε λογικά θα έχουμε αστάθεια αφού η λύση ξεφεύγει. Τώρα αρνητική εντός του κύκλου σημαίνει ότι τείνει στο Σ.Ι, σε κάθε περίπτωση νομίζω ότι ο κύκλος δεν είναι οριακός ασυμπτωτικά ευσταθής.

2γ) Λογικά θα τύχει να έχουμε ένα σημείο ισορροπίας της μορφής (α, 0). Τότε αν πας με Lyapunov θα βγάλεις ότι η V' είναι αρνητικά ημι-ορισμένη και άρα το Σ.Ι είναι απλώς ευσταθές. Όμως αν πας με LaSalle τότε λογικά θα βγάλεις ότι το S = {...} (βλέπε σημειώσεις Apostolf σελ 33) είναι το μεγαλύτερο αμετάβλητο σύνολο και άρα έχουμε ασυμπτ ευσταθεια.






Title: Re: [ΣΑΕ ΙΙ] Θέματα 2017
Post by: princess_of_the_dawn on January 23, 2018, 16:28:49 pm
Ερώτηση

Σε πολλές ενδεικτικές λύσεις έχω δει να αγνοείται η βαρύτητα στην κατάστρωση των εξισώσεων (π.χ. Προοδος 17 ή Φλεβ 17). Το σωστό δεν θα ήταν να μην την αγνοήσουμε;

1) Στις σημειώσεις λέει ότι μια συνάρτηση V είναι αρνητικά ορισμένη αν V > 0 και V(x) != 0 για χ !=0 και V(x) = 0 για χ = 0.
Άρα λογικά το V' = -x2sinx1 δεν είναι αρνητικά ορισμένο αφού μηδενίζεται και για x1 !=0...

2α) Νομίζω πως είναι ασυμπτωτικά ευσταθές

2β) Αν γίνεται θετική εκτός του κύκλου τότε λογικά θα έχουμε αστάθεια αφού η λύση ξεφεύγει. Τώρα αρνητική εντός του κύκλου σημαίνει ότι τείνει στο Σ.Ι, σε κάθε περίπτωση νομίζω ότι ο κύκλος δεν είναι οριακός ασυμπτωτικά ευσταθής.

2γ) Λογικά θα τύχει να έχουμε ένα σημείο ισορροπίας της μορφής (α, 0). Τότε αν πας με Lyapunov θα βγάλεις ότι η V' είναι αρνητικά ημι-ορισμένη και άρα το Σ.Ι είναι απλώς ευσταθές. Όμως αν πας με LaSalle τότε λογικά θα βγάλεις ότι το S = {...} (βλέπε σημειώσεις Apostolf σελ 33) είναι το μεγαλύτερο αμετάβλητο σύνολο και άρα έχουμε ασυμπτ ευσταθεια.

Για τη βαρύτητα, συμφωνώ δε νομίζω να πρέπει να αγνοηθεί.

ευχαριστώ πολύ! :)

2α) πώς το σ.ι είναι ασυμπτωτικά ευσταθές ? Αφού υπάρχει μεγαλυτερο αμετάβλητο σύνολο από το σ.ι στο οποίο τείνουν οι λύσεις της V παράγωγος ....
2β)οχι δε μιλάω για τον κύκλο, μιλάω για το σ.ι όπως και παραπάνω
   αυτό δλδ εντός του κύκλου να είναι ασυμπτωτικά ευσταθές
γ) ναι το μεγαλύτερο αμετάβλητο σύνολο όμως είναι το (α,0) κι εγω μιλάω για την ευστάθεια του (0,0)


Title: Re: [ΣΑΕ ΙΙ] Θέματα 2017
Post by: ENtz on January 23, 2018, 16:31:20 pm
Ας πει κάποιος τη γνωμη του.

Εγώ θα έλεγα το 2 σίγουρα και το 4(με  δισταγμό). Εφ'όσον η V είναι συνάρτηση Lyap σημαίνει ότι ικανοποιεί την προϋπόθεση να ειναι θετική παντού  εκτός από τα σημεία ισορροπίας* όπου μηδενίζεται και αρνητική παράγωγος σημαίνει ότι η συνάρτηση τείνει προς τα σημεία ισορροπίας. Στο διάγραμμα φαίνεται σημεία ισορροπίας (έστω σ.ι: α,-α,β,-β) να ειναι τα σημεία όπου μηδενίζουν τα x1 &x2 άρα θα θέλουμε η f(x) να "τραβάει" τη συνάρτηση προς τα σημεία αυτά, το οποίο το κάνουν τα διανύσματα (2) και (4). Όχι;;;

*θετικά ορισμένη είναι η συνάρτηση η οποία μηδενίζεται μόνο στο 0 ή μόνο στα σ.ι;;
εδιτ: απαντάω στην εαυτή μου: μάλλον μόνο στα σ.ι απλά κάνουμε συνήθως γραμμικοποίηση γι αυτό θεωρούμε ότι μηδενίζεται στο 0

Δες λίγο σελίδα 29 απο τις συνολικες σημειωσεις, λεει η F(x) θα πρεπει να κατευθυνεται στο εσωτερικο της ισοβαρους, αρα σιγουρα οχι το 4. Λογικα ειναι το 3 κ οχι το 2 γτ με το 2 φευγει εκτος του κυκλου ενω εχεις V'<0.


Title: Re: [ΣΑΕ ΙΙ] Θέματα 2017
Post by: princess_of_the_dawn on January 23, 2018, 17:23:51 pm
Δες λίγο σελίδα 29 απο τις συνολικες σημειωσεις, λεει η F(x) θα πρεπει να κατευθυνεται στο εσωτερικο της ισοβαρους, αρα σιγουρα οχι το 4. Λογικα ειναι το 3 κ οχι το 2 γτ με το 2 φευγει εκτος του κυκλου ενω εχεις V'<0.
ναι τώρα που το ξαναλές σα να χεις δίκαιο
απλά μπερδευόμουν επειδή θεωρούσα ότι τα σημεία ισορροπίας είναι αυτά στα οποία η V τέμνει τους άξονες ενώ στην ουσία είναι το (0,0) για να είναι Lyap
βέβαια με μπερδεύει κάπως η ύπαρξη του κύκλου αλλά τί να κάνεις...


Title: Re: [ΣΑΕ ΙΙ] Θέματα 2017
Post by: asamantas on January 23, 2018, 17:50:34 pm
Δες λίγο σελίδα 29 απο τις συνολικες σημειωσεις, λεει η F(x) θα πρεπει να κατευθυνεται στο εσωτερικο της ισοβαρους, αρα σιγουρα οχι το 4. Λογικα ειναι το 3 κ οχι το 2 γτ με το 2 φευγει εκτος του κυκλου ενω εχεις V'<0.

Είπες θα πρεπει να κατευθυνεται στο εσωτερικο της ισοβαρους, αρα και το 2. οχι;


Title: Re: [ΣΑΕ ΙΙ] Θέματα 2017
Post by: link97 on May 17, 2018, 13:47:35 pm
Εχει κανεις ιδεα για το ερωτημα  β ??? :-\
https://prnt.sc/jj0jgn


Title: Re: [ΣΑΕ ΙΙ] Θέματα 2017
Post by: thanospr on June 05, 2018, 13:21:43 pm
Θεματα Σεπτεμβριου 2017.Τα εχει λυσει κανενας;


Title: Re: [ΣΑΕ ΙΙ] Θέματα 2017
Post by: kanou_tom on June 05, 2018, 18:07:06 pm
Έλυσε κάποιος το ερώτημα β Θέμα 2ο Σεπτέμβριος 2017;


Title: Re: [ΣΑΕ ΙΙ] Θέματα 2017
Post by: link97 on June 05, 2018, 20:45:50 pm
Έλυσε κάποιος το ερώτημα β Θέμα 2ο Σεπτέμβριος 2017;

http://helit.org/ece-notes/sae2.pdf#page=67
https://prnt.sc/jr8pmr


Title: Re: [ΣΑΕ ΙΙ] Θέματα 2017
Post by: kanou_tom on June 06, 2018, 15:54:21 pm
Στις εξισωσεις στο θέμα 1ο της προόδου 2017 την βαρύτητα την λαμβάνουμε κανονικά υπόψιν;


Title: Re: [ΣΑΕ ΙΙ] Θέματα 2017
Post by: leukosaraphs! on June 06, 2018, 16:11:27 pm
Στις εξισωσεις στο θέμα 1ο της προόδου 2017 την βαρύτητα την λαμβάνουμε κανονικά υπόψιν;

εγω πιστευω πως ναι


Title: Re: [ΣΑΕ ΙΙ] Θέματα 2017
Post by: trelonoobaki on June 06, 2018, 17:01:53 pm
Στις εξισωσεις στο θέμα 1ο της προόδου 2017 την βαρύτητα την λαμβάνουμε κανονικά υπόψιν;

είμαι 99% σίγουρος ότι ΟΧΙ

edit: pare auto


Title: Re: [ΣΑΕ ΙΙ] Θέματα 2017
Post by: afro on June 06, 2018, 17:54:10 pm
Στις εξισωσεις στο θέμα 1ο της προόδου 2017 την βαρύτητα την λαμβάνουμε κανονικά υπόψιν;
http://www.sharetechnote.com/html/DE_Modeling_Example_SpringMass.html#SingleSpring_SimpleHarmonic
εδώ εξηγεί γιατί δεν το λαμβάνει υπόψιν άμα πάρει ως αναφορά τη θέσηισορροπίας.Εχει και το παράδειγμα από μία πτυχιακή εξέταση με τις αναρτήσεις του αυτόκινήτου  


Title: Re: [ΣΑΕ ΙΙ] Θέματα 2017
Post by: AB_Kingbird on June 06, 2018, 21:10:41 pm
Δεν μπορώ να βρω κάτι σχετικό εδώ (ή δεν έψαξα αρκετά  :P) αλλά γνωρίζεις κανείς τι είναι αυτα που ζητάει στο πρώτο θέμα, ερωτήματα (δ) και (ε) του Σεπτεμβρίου 2017;

Επίσης, στο (γ), απλά λέμε ότι η υλοποίηση είναι ελάχιστη όταν το σύστημα είναι παρατηρήσιμο και ελέγξιμο;

Για το 3ο θέμα, καμία ιδέα γενικά; Προσπάθησα να κάνω κάτι με Lyapunov αλλά δεν μπορώ να βγάλω τίποτα.


Title: Re: [ΣΑΕ ΙΙ] Θέματα 2017
Post by: potis on June 07, 2018, 17:56:52 pm
Στα θέματα Ιουνίου 2017, στο 3α ζητάει τον πίνακα μετασχηματισμού Τ. Ξέρει κάποιος πως το υπολογίζουμε αυτό;


Title: Re: [ΣΑΕ ΙΙ] Θέματα 2017
Post by: κύριος Φασόλης on June 07, 2018, 17:57:13 pm
Για το θεμα 4β του Ιουνιου 17 μπορουμε να πουμε οτι επειδη το ΣΙ ειναι το (0,0) μπορω να γραψω τους ορους με τα τετραγωνα οτι ειναι σχεδον ισοι με κ^2 και να γραμμικοποιησω ετσι το συστημα μου;


Title: Re: [ΣΑΕ ΙΙ] Θέματα 2017
Post by: empargio on June 07, 2018, 18:04:50 pm
Στα θέματα Ιουνίου 2017, στο 3α ζητάει τον πίνακα μετασχηματισμού Τ. Ξέρει κάποιος πως το υπολογίζουμε αυτό;

ειναι νομιζω Τ=M'M^(-1) οπου Μ΄ ο πινακας ελεξιμοτητας της ελεξιμης μορφης και Μ ο πινακας ελεξιμοτητας της αρχικης


Title: Re: [ΣΑΕ ΙΙ] Θέματα 2017
Post by: princess_of_the_dawn on June 07, 2018, 18:09:05 pm
Για το θεμα 4β του Ιουνιου 17 μπορουμε να πουμε οτι επειδη το ΣΙ ειναι το (0,0) μπορω να γραψω τους ορους με τα τετραγωνα οτι ειναι σχεδον ισοι με κ^2 και να γραμμικοποιησω ετσι το συστημα μου;
τί ακριβώς εννοείς?

Στα θέματα Ιουνίου 2017, στο 3α ζητάει τον πίνακα μετασχηματισμού Τ. Ξέρει κάποιος πως το υπολογίζουμε αυτό;

Έχεις ένα σύστημα με πίνακες Α και Β τους οποίους ξέρεις και βρίσκεις ότι το σύστημα είναι ελέγξιμο άρα μπορείς να το φέρεις στην εκμ η οποία έχει πίνακα Α'.
Το χαρακτηριστικό πολυώνυμο του συστήματος που βλεπεις στην εκφώνηση δίνεται από την εξίσωση det(sI-A)=0 και είναι της μορφής s2+as+b ενώ το χπ της εκμ ειναι s2+a1s+a2. Οι δύο μορφές είναι ισοδύναμες αφού η εκμ γίνεται με κατάλληλο ΜΣ ζ=Τχ χ=Τζοπότε εξισώνοντας τα όρους των δύο πολυωνύμων βρίσκεις τα α1, α2 και φτιάχνεις τον Α'. Ο Β' στην εκμ είναι πάντα [1 0...0]Τ και με τα Α', Β' φτιάχνεις το Μ'. Ο μη ιδιόμορφος Τ= Μ'Μ-1 και τον Μ προφανώς τον έχεις βρει πρώτον πρώτον.

'Εχει απαντηθεί ποοοοοοοολύ πίσω αν δεν απατώμεν.

εδιτ:με πρόλαβαν :P


Title: Re: [ΣΑΕ ΙΙ] Θέματα 2017
Post by: κύριος Φασόλης on June 07, 2018, 18:13:55 pm
τί ακριβώς εννοείς?


There you go


Title: Re: [ΣΑΕ ΙΙ] Θέματα 2017
Post by: potis on June 07, 2018, 18:16:33 pm
τί ακριβώς εννοείς?

Έχεις ένα σύστημα με πίνακες Α και Β τους οποίους ξέρεις και βρίσκεις ότι το σύστημα είναι ελέγξιμο άρα μπορείς να το φέρεις στην εκμ η οποία έχει πίνακα Α'.
Το χαρακτηριστικό πολυώνυμο του συστήματος που βλεπεις στην εκφώνηση δίνεται από την εξίσωση det(sI-A)=0 και είναι της μορφής s2+as+b ενώ το χπ της εκμ ειναι s2+a1s+a2. Οι δύο μορφές είναι ισοδύναμες αφού η εκμ γίνεται με κατάλληλο ΜΣ ζ=Τχ χ=Τζοπότε εξισώνοντας τα όρους των δύο πολυωνύμων βρίσκεις τα α1, α2 και φτιάχνεις τον Α'. Ο Β' στην εκμ είναι πάντα [1 0...0]Τ και με τα Α', Β' φτιάχνεις το Μ'. Ο μη ιδιόμορφος Τ= Μ'Μ-1 και τον Μ προφανώς τον έχεις βρει πρώτον πρώτον.

'Εχει απαντηθεί ποοοοοοοολύ πίσω αν δεν απατώμεν.

εδιτ:με πρόλαβαν :P

Ευχαριστώ πάρα πολύ!!!


Title: Re: [ΣΑΕ ΙΙ] Θέματα 2017
Post by: princess_of_the_dawn on June 07, 2018, 18:26:16 pm
There you go

Εν προκειμένω βγαίνει σωστό δε ξέρω όμως αν είναι συμπτωματικό. Προσωπικά δε θα το ρίσκαρα αφού η Ιακωβιανή βγαίνει πανεύκολα εδώ.
Αλλά ποιά η φυσική σημασία της υπόθεσής σου? Θέλω να πω, αν δεν υπήρχε ο πολλαπλασιαστικός παράγοντας χ1 πχ και είχες απλά κ21 θα υπέθετες ότι ισούται με κ2 ? Αν ναι, το 1ο στοιχείο της ιακωβιανής θα έβγαινε 0 ενώ κανονικά θα έπρεπε να βγεί 1.

Πάντως η υπόθεση σου είναι κάτι σαν "κοντά στο σ.ι. το σύστημα παίρνει τιμές πάνω σε οριακό κύκλο" το οποίο αν το σημείο ισορροπίας είναι ασταθές μάλλον ισχύει, δηλ. αν υπάρχει ε<κ2 για την οποία η λυαπ να είναι θετική τότε όντως έχει νόημα, αν όμως ήταν ευσταθές ΥΠΟΘΕΤΩ πως δε θα ίσχυε.  :-\ :-\ #confused


Title: Re: [ΣΑΕ ΙΙ] Θέματα 2017
Post by: κύριος Φασόλης on June 07, 2018, 18:34:04 pm
Εν προκειμένω βγαίνει σωστό δε ξέρω όμως αν είναι συμπτωματικό. Προσωπικά δε θα το ρίσκαρα αφού η Ιακωβιανή βγαίνει πανεύκολα εδώ.
Αλλά ποιά η φυσική σημασία της υπόθεσής σου? Θέλω να πω, αν δεν υπήρχε ο πολλαπλασιαστικός παράγοντας χ1 πχ και είχες απλά κ21 θα υπέθετες ότι ισούται με κ2 ? Αν ναι, το 1ο στοιχείο της ιακωβιανής θα έβγαινε 0 ενώ κανονικά θα έπρεπε να βγεί 1.

Πάντως η υπόθεση σου είναι κάτι σαν "κοντά στο σ.ι. το σύστημα παίρνει τιμές πάνω σε οριακό κύκλο" το οποίο αν το σημείο ισορροπίας είναι ασταθές μάλλον ισχύει, δηλ. αν υπάρχει ε<κ2 για την οποία η λυαπ να είναι θετική τότε όντως έχει νόημα, αν όμως ήταν ευσταθές ΥΠΟΘΕΤΩ πως δε θα ίσχυε.  :-\ :-\ #confused

γενικα το σκεπτικο μου το βασισα στο παραδειγμα του ανεστραμμενου εκκρεμους που εχει κανει. Που εχει ενα sinx1 και επειδη το x1 του στο ΣΙ ειναι το 0 λεει μπορω να υποθεσω οτι sinx1 = x1.

Εδω η F ποια ειναι; γιατι σε προηγουμενες σελιδες λενε μια οτι ειναι η πρωτη εξισωση (δηλαδη του χ1_dot) και μια οτι ειναι το συστημα και των 2.


Title: Re: [ΣΑΕ ΙΙ] Θέματα 2017
Post by: princess_of_the_dawn on June 07, 2018, 18:40:26 pm
Εννοείς για την Ιακωβιανή; Το σύστημα και των δύο, σκέψου ότι στη σειρά i έχουμε δFi και στη στήλη j έχουμε /δzj.

Aυτό με το εκκρεμές το  υποθέτεις χαριν ευκολίας εδώ δε ξέρω αν μπορείς να το κάνεις αλλά ας πει κανας πιο σοφός τη γνώμη του.


Title: Re: [ΣΑΕ ΙΙ] Θέματα 2017
Post by: κύριος Φασόλης on June 07, 2018, 18:45:02 pm
Εννοείς για την Ιακωβιανή; Το σύστημα και των δύο, σκέψου ότι στη σειρά i έχουμε δFi και στη στήλη j έχουμε /δzj.

Aυτό με το εκκρεμές το  υποθέτεις χαριν ευκολίας εδώ δε ξέρω αν μπορείς να το κάνεις αλλά ας πει κανας πιο σοφός τη γνώμη του.

Ωραια, ενιωσα τι λες σε ευχαριστω πολυ!


Title: Re: [ΣΑΕ ΙΙ] Θέματα 2017
Post by: princess_of_the_dawn on June 07, 2018, 20:15:58 pm
Δεν έχω ιδέα αν είναι σωστό. Θέμα 3 του Σεπτέμβρη 2017



Title: Re: [ΣΑΕ ΙΙ] Θέματα 2017
Post by: AB_Kingbird on June 07, 2018, 20:20:42 pm
Ορίστε και το θέμα 1 του Σεπτέμβρη 2017 το οποίο ανεβάζω κυρίως λόγω των 2 τελευταίων ερωτημάτων για τα οποία δεν είμαι (σχεδόν καθόλου) σίγουρος για τη λύση τους.


Title: Re: [ΣΑΕ ΙΙ] Θέματα 2017
Post by: princess_of_the_dawn on June 07, 2018, 20:28:49 pm
Άρα λες ότι ελάχιστο πλήθος ενεργοποιητών =2 (αφού α!=0) και αισθητήρων =1 (αφού το β μπορεί να είναι 0)?
Κι εγώ αυτό είπα. :P


Title: Re: [ΣΑΕ ΙΙ] Θέματα 2017
Post by: AB_Kingbird on June 07, 2018, 20:37:03 pm
Δεν έχω ιδέα αν είναι σωστό αλλά παρε.

Μπράβο σου!
Άρα λες ότι ελάχιστο πλήθος ενεργοποιητών =2 (αφού α!=0) και αισθητήρων =1 (αφού το β μπορεί να είναι 0)?
Κι εγώ αυτό είπα. :P

Και μπράβο μας! Ναι αυτό εννοούσα ότι από τη στιγμή που δεν γίνεται να είναι μηδέν, υπάρχει, άρα 2 ελεγκτές και 1 αισθητήρας το ελάχιστο.

Κάτι ακόμα που μπορεί να είναι και χαζο: Στην πρώτη λύση που ανέβασες, στην παράγωγο dz2'/dz2, πως ακριβώς παραγώγισες το |z2|?


Title: Re: [ΣΑΕ ΙΙ] Θέματα 2017
Post by: princess_of_the_dawn on June 07, 2018, 20:43:42 pm
εχμμ το θεωρησα ίσο  με z2 δηλ σα να εχω ζ^2
επίσης τώρα που ξαναβλέπω τη λύση μαλλον ειναι λάθος μισό
 εδιτ: ναι δε μπορείς να θεωρήσεις αυτό που θεώρησα οπότε αυτό που γράφω παρακάτω είναι το σωστό

εδιτ: ε χμμμ δε ξέρω δε μπορω να σκεφτώ τι αλλο να κανω με τo |z2| απλά ίσως μπορείς να το αποφύγεις ορίζοντας στο αρχικό σύστημα Lyap την V=(k/2M)(x1-(Mg)/k)2+(1/2)x22
 η οποία είναι πάλι αρνητικά ημιορισμένη (για χ1*=c) και μετά κάνεις LaSalle για το σημείο ισορροπίας Μg/k δηλ υποθέτεις πως η λύση του χ1 για V'=0 είναι διάφορη αυτού του αριθμού.


Title: Re: [ΣΑΕ ΙΙ] Θέματα 2017
Post by: empargio on June 08, 2018, 10:49:23 am
Ορίστε και το θέμα 1 του Σεπτέμβρη 2017 το οποίο ανεβάζω κυρίως λόγω των 2 τελευταίων ερωτημάτων για τα οποία δεν είμαι (σχεδόν καθόλου) σίγουρος για τη λύση τους.

στα α και β σε ρωταει για ποιες τιμες ΔΕΝ ειναι ελεξιμο κ παρατηρσιμο αρα νομιζω πρεπει να πεις det=0 για τους πιανκες ελεξιμοτητας και παρατρισιμοτητας


Title: Re: [ΣΑΕ ΙΙ] Θέματα 2017
Post by: κύριος Φασόλης on June 08, 2018, 11:21:32 am
στα α και β σε ρωταει για ποιες τιμες ΔΕΝ ειναι ελεξιμο κ παρατηρσιμο αρα νομιζω πρεπει να πεις det=0 για τους πιανκες ελεξιμοτητας και παρατρισιμοτητας

exactly


Title: Re: [ΣΑΕ ΙΙ] Θέματα 2017
Post by: Alex_corfu on July 29, 2018, 12:52:25 pm
Ανέβασα λύσεις θεμάτων από 2014-2017 στα downloads σε περίπτωση που κάποιος δεν τα έχει προσέξει, με κάθε επιφύλαξη.


Title: Re: [ΣΑΕ ΙΙ] Θέματα 2017
Post by: Florence on September 08, 2018, 23:50:13 pm
Στις λυσεις στα downloads για το ερωτημα δ 1ο θεμα Σεπτεμβριου 17, προκειμενου να βρει το ελαχιστο πληθος ενεργοποιητων στον πινακα Β=[α 1]^-1 μηδενιζει το στοιχειο με τιμη 1 επειδη δεν μπορει να μηδενισει το α(χανεται η ελεγξιμοτητα)..γινεται αυτο? δεν αλλαζει ετσι το συστημα ομως?


Title: Re: [ΣΑΕ ΙΙ] Θέματα 2017
Post by: leukosaraphs! on September 09, 2018, 00:37:27 am
Στις λυσεις στα downloads για το ερωτημα δ 1ο θεμα Σεπτεμβριου 17, προκειμενου να βρει το ελαχιστο πληθος ενεργοποιητων στον πινακα Β=[α 1]^-1 μηδενιζει το στοιχειο με τιμη 1 επειδη δεν μπορει να μηδενισει το α(χανεται η ελεγξιμοτητα)..γινεται αυτο? δεν αλλαζει ετσι το συστημα ομως?

εχεις δικιο. Ωστοσο δεν μπορεις να ξερεις τι εχει στο μυαλο του ο ροβι, και ετσι θα μπορουσες να σημειωσεις οτι:

"Αν μηδενισω το α εχω το ταδε προβλημα, αν ωστοσο μου δινοταν η δυνατοτητα να πειραξω το αλλο στοιχειο του πινακα Β, τοτε θα μπορουσα να εχω εναν ενεργοποιητη."

Υγ. εγω προσωπικα θα απαντουσα οτι το ελαχιστο πληθος = 2.


Title: Re: [ΣΑΕ ΙΙ] Θέματα 2017
Post by: Florence on September 09, 2018, 12:01:21 pm
ευχαριστω, αλλα διδαχτηκαμε φετος καθολου για ενεργοποιητες και αισθητηρες? δε θυμαμαι να το λεει σε καποια διαλεξη του ο ροβι ουτε το βρηκα στις σημειωσεις...και αν ναι σε ποιο κεφαλαιο?


Title: Re: [ΣΑΕ ΙΙ] Θέματα 2017
Post by: MrRobot on September 09, 2018, 12:48:50 pm
Τα είχε αναφέρει στο πρώτο μάθημα. Μετά τα θεωρεί γνωστά υποθέτω.


Title: Re: [ΣΑΕ ΙΙ] Θέματα 2017
Post by: Ζak Μckracken on September 10, 2018, 22:40:20 pm
Σεπτεμβριος 2017..... θεμα 1γ)
Ποτε ένα σύστημα έχει ελάχιστη υλοποίηση;;;;


Title: Re: [ΣΑΕ ΙΙ] Θέματα 2017
Post by: leukosaraphs! on September 10, 2018, 22:47:48 pm
Σεπτεμβριος 2017..... θεμα 1γ)
Ποτε ένα σύστημα έχει ελάχιστη υλοποίηση;;;;

αρχικα λιγο περισσοτερη προσοχη που ποσταρουμε... τοση δουλεια ριξαμε για να γινουν τα τοπικ ευαναγνωστα! Τα τοπικ ειναι ανα χρονια!

το συστημα εχει ελαχιστη υλοποιηση, οταν ειναι ελεγξιμο και παρατηρησιμο.


Title: Re: [ΣΑΕ ΙΙ] Θέματα 2017
Post by: Ragnar Lothbrok on September 11, 2018, 13:41:16 pm
Σεπτέμβριος 2017 θέμα τρίτο έχουμε καμία λύση; δεν μπορώ να βρω τη lyapunov


Title: Re: [ΣΑΕ ΙΙ] Θέματα 2017
Post by: Ragnar Lothbrok on September 11, 2018, 15:31:04 pm
Υπάρχει Σεπτεμβριος 2017 3ο θέμα λυμένο;


Title: Re: [ΣΑΕ ΙΙ] Θέματα 2017
Post by: Elina97 on September 12, 2018, 21:01:53 pm
Υπάρχει Σεπτεμβριος 2017 3ο θέμα λυμένο;
Στην προηγούμενη σελίδα  ;)


Title: Re: [ΣΑΕ ΙΙ] Θέματα 2017
Post by: potis on September 13, 2018, 14:59:40 pm
Στο 4γ Ιούνιο 2017 που παίρνουμε περιπτώσεις για το πρόσημο της Lyaponov, γιατί καταλήγουμε στο συμπέρασμα της αστάθειας; Δεν θα μπορούσαμε να πούμε πως το σύστημα είναι τοπικά ευσταθές στην περίπτωση που η Lyapunov βγαίνει αρνητικά ορισμένη;


Title: Re: [ΣΑΕ ΙΙ] Θέματα 2017
Post by: abadasa13 on September 13, 2018, 16:22:29 pm
Ιούνιος 17: Θέμα 5. Για να καταλάβω, από τη στιγμή που δείξουμε ότι το σύστημα είναι ελέγξιμο και βρούμε την τιμή του κ2 για να ικανοποιείται η προϋπόθεση του ζ = 1, δε χρειάζεται να δείξουμε ότι:

α) Το (0,0) είναι σημείο ισορροπίας; (ντάξει αυτό κλάιν)

β) Το συγκεκριμένο Σ.Ι. είναι ασυμπτωτικά ευσταθές;

Η ελεγξιμότητα συμπεριλαμβάνει την ασυμπτωτική ευστάθεια ΟΛΩΝ των Σ.Ι.;

και κάτι πιθανόν πιο χαζό: Η ελεγξιμότητα απ'όσα έχω καταλάβει μας βοηθάει να ελέγξουμε την ταχύτητα σύγκλισης στην τελική τιμή. Μπορούμε να μετατοπίσουμε μέσω ελεγκτή το-τα Σ.Ι.;

Όλα αυτά στα πλαίσια της λύσης που διάβασα στο "SAEII_Themata_Lyseis_2014_2017.pdf" στη σελ. 42.


Title: Re: [ΣΑΕ ΙΙ] Θέματα 2017
Post by: abadasa13 on September 13, 2018, 17:44:39 pm
Στο 4γ Ιούνιο 2017 που παίρνουμε περιπτώσεις για το πρόσημο της Lyaponov, γιατί καταλήγουμε στο συμπέρασμα της αστάθειας; Δεν θα μπορούσαμε να πούμε πως το σύστημα είναι τοπικά ευσταθές στην περίπτωση που η Lyapunov βγαίνει αρνητικά ορισμένη;

Αν έχω καταλάβει σωστά τι ακριβώς ρωτάς -κι από τα όσα έχω καταλάβει-, βγαίνει ότι είναι ασταθές γιατί το αρνητικό πρόσημο της V εξαρτάται από τις τιμές των x1 και x2 σε συνάρτηση με το κ. Ουσιαστικά -το λέω με επιφύλαξη- αν πάρεις τιμές χ1 χ2 κοντά στο Σ.Ι. (αλλά και οπουδήποτε αλλού) θα καταλήγουν (το πώς καταλήγουν φαίνεται από τις παραγώγους) στον οριακό κύκλο και όχι στο Σ.Ι. σου. Μόνο όταν χ1=χ2=0 μένει εκεί το σύστημα το οποιό πρακτικά σημαίνει ασταθές Σ.Ι.

Αν κάνω λάθος ας με διορθώσει κάποιος.


Title: Re: [ΣΑΕ ΙΙ] Θέματα 2017
Post by: Florence on September 14, 2018, 00:22:26 am
Ιούνιος 17: Θέμα 5. Για να καταλάβω, από τη στιγμή που δείξουμε ότι το σύστημα είναι ελέγξιμο και βρούμε την τιμή του κ2 για να ικανοποιείται η προϋπόθεση του ζ = 1, δε χρειάζεται να δείξουμε ότι:

α) Το (0,0) είναι σημείο ισορροπίας; (ντάξει αυτό κλάιν)

β) Το συγκεκριμένο Σ.Ι. είναι ασυμπτωτικά ευσταθές;

Η ελεγξιμότητα συμπεριλαμβάνει την ασυμπτωτική ευστάθεια ΟΛΩΝ των Σ.Ι.;

και κάτι πιθανόν πιο χαζό: Η ελεγξιμότητα απ'όσα έχω καταλάβει μας βοηθάει να ελέγξουμε την ταχύτητα σύγκλισης στην τελική τιμή. Μπορούμε να μετατοπίσουμε μέσω ελεγκτή το-τα Σ.Ι.;

Όλα αυτά στα πλαίσια της λύσης που διάβασα στο "SAEII_Themata_Lyseis_2014_2017.pdf" στη σελ. 42.


+1


Title: Re: [ΣΑΕ ΙΙ] Θέματα 2017
Post by: dounavis on January 06, 2019, 22:08:54 pm
Φεβρουάριος 2017?? Για το α κτλβαίνω ότι θα χρησιμοποιήσουμε την V(x) =1/2x^Tx αλλά στη συνέχεια τι? :D


Title: Re: [ΣΑΕ ΙΙ] Θέματα 2017
Post by: dounavis on January 17, 2019, 12:45:38 pm
Φεβρουάριος 2017 κανείς???


Title: Re: [ΣΑΕ ΙΙ] Θέματα 2017
Post by: dounavis on January 22, 2019, 20:35:12 pm
Φεβρουάριος 2017 κανείς?????????


Title: Re: [ΣΑΕ ΙΙ] Θέματα 2017
Post by: kostasvou on January 23, 2019, 23:25:05 pm
Φεβρουάριος 2017 κανείς?????????
Αν παρεις V=1/2x^2, τοτε V' = x^2*[a-k(b+f(x)]. Μετα πρεπει να βρουμε Κ τετοιο ωστε να ειναι ολη η αγκυλη αρνητικη. Αλλα με μπερδευει επειδη δεν ξερουμε προσημο για το β...


Title: Re: [ΣΑΕ ΙΙ] Θέματα 2017
Post by: Chandler on January 24, 2019, 00:22:53 am
Βάλε V=(1/2*|b|)*x^2


Title: Re: [ΣΑΕ ΙΙ] Θέματα 2017
Post by: dounavis on January 24, 2019, 03:26:38 am
ας ανεβασει καποιος αν το εχει λυσει το θεμα pleasee.Όπως και το 2


Title: Re: [ΣΑΕ ΙΙ] Θέματα 2017
Post by: kostasvou on January 24, 2019, 19:52:01 pm
Έχω βέβαια μια ένσταση για το 1/(b+c) αλλά είναι το καλύτερο που μπόρεσα να βγάλω (εγώ αυτό θα έγραφα :P ). Νομίζω η συνάρτηση V=(x^2)/(2|b|) δεν βοήθησε σε αυτό το ερώτημα, πάλι στην ίδια συνθήκη καταλήγεις.


Title: Re: [ΣΑΕ ΙΙ] Θέματα 2017
Post by: Ζak Μckracken on January 25, 2019, 01:06:44 am
Άντε και ο Θεός μαζί μας αύριο....


Title: Re: [ΣΑΕ ΙΙ] Θέματα 2017
Post by: pesto80 on June 07, 2019, 13:42:21 pm
Δεν έχω ιδέα αν είναι σωστό. Θέμα 3 του Σεπτέμβρη 2017



Η ασκηση ζηταει ολικα ασυμπτωτικα ευσταθες. Η γραμμικοποιηση γυρω απο το ΣΙ ωστοσο σε αναγκαζει τα αποτελεσματα σου να αφορουν αποκλειστικα και μονο την τοπικη ευσταθεια.


Title: Re: [ΣΑΕ ΙΙ] Θέματα 2017
Post by: pesto80 on June 07, 2019, 18:29:01 pm
Υπάρχει μήπως κάποια σωστή λύση για το σεπτ 17 θέμα 3 αρα;;;


Title: Re: [ΣΑΕ ΙΙ] Θέματα 2017
Post by: ροζ θορυβος on June 07, 2019, 18:38:28 pm
Υπάρχει μήπως κάποια σωστή λύση για το σεπτ 17 θέμα 3 αρα;;;
+1


Title: Re: [ΣΑΕ ΙΙ] Θέματα 2017
Post by: evrichon on June 08, 2019, 21:32:52 pm
Καλησπέρα παραθέτω την λύση για το α) μέρος, νομίζω ότι είναι σωστή.
Παρατήρηση 1) το σημείο ισορροπιας εμφανιζεται στην lyapunov
                      2)Το θεωρημα lasalle το εφαρμοζω γυρο απο το σημειο ισορροπιας που δεν ειναι το μηδεν οπως λεει στην θεωρια αλλα με αλλαγη μεταβλητης
                          λογικα ειναι το ιδιο



Title: Re: [ΣΑΕ ΙΙ] Θέματα 2017
Post by: evrichon on June 08, 2019, 21:47:04 pm
.. και το β) μερος, αν καποιος βρει λαθος ας πει


Title: Re: [ΣΑΕ ΙΙ] Θέματα 2017
Post by: mgavalet on June 09, 2019, 01:05:54 am
Εγώ πήρα πιο απλή lyapunov : V = (k/2)*(x1-gM/k)^2 + (M/2)*x2^2 και βγαινει μια χαρα
στο β πηρα u = c2x2|x2| -gM +r (αφου στις ΕΚ προκυπτει u/M) δε βαζω k1 k2 αφου δε σβηνεις τα χ1 χ2 απ την αρχικη, γιατι να βαλεις κ αλλα?


Αυτη η Λιαπουνοφ δεν ειναι θετικα ορισμενη . αρα δεν μπορουμε να την χρησιμοποιησουμε .


Title: Re: [ΣΑΕ ΙΙ] Θέματα 2017
Post by: pesto80 on June 09, 2019, 22:45:39 pm
Καλησπέρα παραθέτω την λύση για το α) μέρος, νομίζω ότι είναι σωστή.
Παρατήρηση 1) το σημείο ισορροπιας εμφανιζεται στην lyapunov
                      2)Το θεωρημα lasalle το εφαρμοζω γυρο απο το σημειο ισορροπιας που δεν ειναι το μηδεν οπως λεει στην θεωρια αλλα με αλλαγη μεταβλητης
                          λογικα ειναι το ιδιο



η λιαπουνοβ σου δεν βγαζει καποιο νοημα ως προς το προσημο της. ουτε αρνητικη ουτε αρνητικα ημιορισμενη.... μπορεις να εξηγησεις πως το σκεφτεσαι ; ;)


Title: Re: [ΣΑΕ ΙΙ] Θέματα 2017
Post by: MrRobot on June 10, 2019, 00:15:38 am
Υπάρχει μήπως κάποια σωστή λύση για το σεπτ 17 θέμα 3 αρα;;;

Μπορείς να γράψεις τις εξισώσεις κατάστασης χρησιμοποιώντας ως x1 το y - Mg/k και x2 το ydot. Τότε θα έχεις xdot = Ax + δ(x2). Αν πάρεις ως Lyapunov την V = xTPx, με τον P θετικά ορισμένο μπορείς κάνοντας τις πράξεις να βγάλεις Vdot = -xTQx + 2δT(x2)Px. Κάνοντας τις πράξεις για τον τελευταίο όρο μπορεί κανείς να δει ότι με σωστή επιλογή του P (p12 = p21 = 0) η Vdot προκύπτει αρνητικά ορισμένη.

Για το δεύτερο ερώτημα κάνε γραμμικοποίηση μέσω ανάδρασης.


Title: Re: [ΣΑΕ ΙΙ] Θέματα 2017
Post by: eleftheria_94 on June 10, 2019, 02:31:47 am
Μπορείς να γράψεις τις εξισώσεις κατάστασης χρησιμοποιώντας ως x1 το y - Mg/k και x2 το ydot. Τότε θα έχεις xdot = Ax + δ(x2). Αν πάρεις ως Lyapunov την V = xTPx, με τον P θετικά ορισμένο μπορείς κάνοντας τις πράξεις να βγάλεις Vdot = -xTQx + 2δT(x2)Px. Κάνοντας τις πράξεις για τον τελευταίο όρο μπορεί κανείς να δει ότι με σωστή επιλογή του P (p12 = p21 = 0) η Vdot προκύπτει αρνητικά ορισμένη.
Χρησιμοποιώντας αυτό το x1 κάνεις γραμμικοποίηση γύρω από το Σ.Ι., οπότε δεν μπορείς να ψάξεις ΟΛΙΚΗ ευστάθεια ανεξάρτητα με τι επιλογή V και Vdot κάνεις μετά. Έτσι δεν είναι? Διορθώστε με αν κάνω λάθος...

Σορυ γενικά έκανα ένα λάθος στις σταθερες στον πρώτο όρο αλλα η λογική παραμένη η ιδια νομιζω τώρα ειναι πιο ξεκαθαρο.
Συμφωνώ, κι εγώ αυτή κατέληξα οτι είναι η πιο σωστή λύση, αν και θεωρώ ότι ίσως υπάρχει κάποια θεωρητική τρύπα στις απαιτήσεις του θεωρήματος La Salle.


Title: Re: [ΣΑΕ ΙΙ] Θέματα 2017
Post by: MrRobot on June 10, 2019, 14:19:04 pm
Χρησιμοποιώντας αυτό το x1 κάνεις γραμμικοποίηση γύρω από το Σ.Ι., οπότε δεν μπορείς να ψάξεις ΟΛΙΚΗ ευστάθεια ανεξάρτητα με τι επιλογή V και Vdot κάνεις μετά. Έτσι δεν είναι? Διορθώστε με αν κάνω λάθος...

Δεν κάνω καμιά γραμμικοποίηση. Απλά μετακινώ το ΣΙ.



Title: Re: [ΣΑΕ ΙΙ] Θέματα 2017
Post by: pesto80 on June 10, 2019, 15:59:13 pm
Χρησιμοποιώντας αυτό το x1 κάνεις γραμμικοποίηση γύρω από το Σ.Ι., οπότε δεν μπορείς να ψάξεις ΟΛΙΚΗ ευστάθεια ανεξάρτητα με τι επιλογή V και Vdot κάνεις μετά. Έτσι δεν είναι? Διορθώστε με αν κάνω λάθος...


Δεν κάνω καμιά γραμμικοποίηση. Απλά μετακινώ το ΣΙ.

Εγώ πήρα πιο απλή lyapunov : V = (k/2)*(x1-gM/k)^2 + (M/2)*x2^2 και βγαινει μια χαρα


Αυτη η Λιαπουνοφ δεν ειναι θετικα ορισμενη . αρα δεν μπορουμε να την χρησιμοποιησουμε .

οπως αποδειχθηκε στην εξεταση σημερα σημερα γινεται και παραγινεται. shout out to Mr.Robot!


Title: Re: [ΣΑΕ ΙΙ] Θέματα 2017
Post by: pesto80 on August 24, 2019, 20:15:36 pm
Μπορείς να γράψεις τις εξισώσεις κατάστασης χρησιμοποιώντας ως x1 το y - Mg/k και x2 το ydot. Τότε θα έχεις xdot = Ax + δ(x2). Αν πάρεις ως Lyapunov την V = xTPx, με τον P θετικά ορισμένο μπορείς κάνοντας τις πράξεις να βγάλεις Vdot = -xTQx + 2δT(x2)Px. Κάνοντας τις πράξεις για τον τελευταίο όρο μπορεί κανείς να δει ότι με σωστή επιλογή του P (p12 = p21 = 0) η Vdot προκύπτει αρνητικά ορισμένη.

Για το δεύτερο ερώτημα κάνε γραμμικοποίηση μέσω ανάδρασης.

για αυτο το θεμα πηρα μια πιο by the book προσεγγιση (σεπτ 17-θεμα 3α). δεν ξερω. τι λετε;


Title: Re: [ΣΑΕ ΙΙ] Θέματα 2017
Post by: Argirios on August 26, 2019, 15:30:00 pm
Φεβρουαριου 17 εχει κανες λυμενα?
sent from mTHMMY (https://play.google.com/store/apps/details?id=gr.thmmy.mthmmy) 


Title: Re: [ΣΑΕ ΙΙ] Θέματα 2017
Post by: Thunderlord on June 17, 2020, 17:10:16 pm
Για το 2ο θέμα της Προόδου 17:

Θεωρώ πως η σωστή απάντηση είναι σίγουρα το 3ο, καθώς θα σχηματίζει αμβλεία γωνία η F με την V'. Το 1ο νομίζω είναι λάθος, αλλά το 2ο και το 4ο δεν ξέρω.

Έχει κανείς καμιά ιδέα;


Title: Re: [ΣΑΕ ΙΙ] Θέματα 2017
Post by: Exotic on June 20, 2020, 16:36:10 pm
Για το 2ο θέμα της Προόδου 17:

Θεωρώ πως η σωστή απάντηση είναι σίγουρα το 3ο, καθώς θα σχηματίζει αμβλεία γωνία η F με την V'. Το 1ο νομίζω είναι λάθος, αλλά το 2ο και το 4ο δεν ξέρω.

Έχει κανείς καμιά ιδέα;

Λοιπόν αν και δεν το έχω διαβάσει και πολύ καλά θεωρώ ότι το 2ο θα ήταν λάθος καθώς είναι εκτος του κύκλου r άρα λογικά σε αυτή την περίπτωση η V' θα ήταν θετική και το ίδιο λογικά ισχυεί και για το 1ο.

Μετα για το 4ο ετσί οπως το βλέπω μάλλον το απορρίπτεις γιατί δεν σχηματίζει αμβλεία γωνια με την V'.
Αν έχει κάποιος αλλος καμία καλύτερη ιδεα ας πει...


Title: Re: [ΣΑΕ ΙΙ] Θέματα 2017
Post by: Thunderlord on June 20, 2020, 16:45:17 pm
Λοιπόν αν και δεν το έχω διαβάσει και πολύ καλά θεωρώ ότι το 2ο θα ήταν λάθος καθώς είναι εκτος του κύκλου r άρα λογικά σε αυτή την περίπτωση η V' θα ήταν θετική και το ίδιο λογικά ισχυεί και για το 1ο.

Μετα για το 4ο ετσί οπως το βλέπω μάλλον το απορρίπτεις γιατί δεν σχηματίζει αμβλεία γωνια με την V'.
Αν έχει κάποιος αλλος καμία καλύτερη ιδεα ας πει...

Γενικά ακόμα παλεύω να βρω σωστή άκρη. Βλέποντας και τη συζήτηση για τα θέματα του 20 (https://www.thmmy.gr/smf/index.php?topic=75838.0), όπου έβαλε το ίδιο θέμα, καταλήγω μάλλον στο 3,4.

Το 4 γιατί λες πως δεν σχεδιάζει αμβίλα γωνία;


Title: Re: [ΣΑΕ ΙΙ] Θέματα 2017
Post by: Exotic on June 20, 2020, 17:07:24 pm
Γενικά ακόμα παλεύω να βρω σωστή άκρη. Βλέποντας και τη συζήτηση για τα θέματα του 20 (https://www.thmmy.gr/smf/index.php?topic=75838.0), όπου έβαλε το ίδιο θέμα, καταλήγω μάλλον στο 3,4.

Το 4 γιατί λες πως δεν σχεδιάζει αμβίλα γωνία;
Νομίζω ότι η εφαπτομενη για να σχεδιασουμε το κάθετο δίάνυσμα της dV/dx ειναι πάνω στην έλλειψη οπότε ετσι οπώς το βλέπω στο σχημα  θεωρώ οτι η γωνία δεν ειναι αμβλεια..αυτη ειναι η σκεψη μου αλλα δεν ξερω αν ειναι σωστη..


edit:Επισης τωρα που ξανα ειδα λιγο το βιντεο 3_4 λεπτο 13,20 περιπου λεει οτι το διανυσμα της F  πρεπει να ειναι εντος της ισοβαρους καμπυλης δλδ στην περιπτωση μας εντος της ελλειψης οποτε ισως και για αυτο..


Title: Re: [ΣΑΕ ΙΙ] Θέματα 2017
Post by: Kkiller_97 on July 08, 2020, 18:20:00 pm
ΦΕΒ 17 θέμα 2 ποιές είναι τελικά οι σωστές εξισώσεις?


Title: Re: [ΣΑΕ ΙΙ] Θέματα 2017
Post by: pesto80 on September 07, 2020, 18:34:09 pm
για αυτο το θεμα πηρα μια πιο by the book προσεγγιση (σεπτ 17-θεμα 3α). δεν ξερω. τι λετε;

σε συνέχεια του παραπάνω, στο Σεπτ 17 - θεμα 3.β δεν μπορούμε απλά να βαλουμε για ελεγκτη το u = c2/M * x2 * |x2| + v , οπου v η νεα εισοδος και αρα απλα να μας βγει το συστημα γραμμικο;;; (βγαινει οτι εχει και ευσταθεια).


Title: Re: [ΣΑΕ ΙΙ] Θέματα 2017
Post by: Caterpillar on June 27, 2021, 16:38:35 pm
Φεβρουαριου 17 εχει κανες λυμενα?
sent from mTHMMY (https://play.google.com/store/apps/details?id=gr.thmmy.mthmmy)  
Τα έχει κανένας λυμένα?
Δεν τα έχω δει σε κανένα αρχείο.


Title: Re: [ΣΑΕ ΙΙ] Θέματα 2017
Post by: Πατερ Ημμυων on June 27, 2021, 19:00:27 pm
λογικά λαθος θα είναι ...


Title: Re: [ΣΑΕ ΙΙ] Θέματα 2017
Post by: Caterpillar on June 27, 2021, 19:32:33 pm
λογικά λαθος θα είναι ...
ωραίο όνομα αρχείου, το θέμα 2 και εγώ έτσι το έχω κάνει, αλλά δεν απάντησα και εγώ σε μια ερώτηση που έχει στο τέλος.
Θεμα 1. Το c μπορούμε να το χρησιμοποιήσουμε στον ελεκτή μας? Γιατί δεν λέει κάπου ότι είναι γνωστό.

edit: Θέμα 1) α) βάζουμε u = 0 για να βγει αυτό που θέλει.